Fonction $\Gamma$

Généralités

($\star\star$) Correction

Pour $n\in \N^*$, on pose $u_n=\dsum_{k=1}^n\frac{1}{k}-\ln(n)$.

  1. Montrer que la suite $(u_n)$ est strictement décroissante.
  2. Montrer que, pour tout $n\in \N^*$, $u_n>0$.
  3. En déduire que $(u_n)_n$ converge.
Correction

  1. Soit $n\in \N^*$, on a $$ \begin{array}{lcl} u_{n+1}-u_n&=&\dsum_{k=1}^{n+1}\frac{1}{k}-\ln(n+1)-\dsum_{k=1}^n\frac{1}{k}+\ln(n)\\ &&\\ &=&\dsp \dfrac{1}{n+1}-\ln(n+1)+\ln(n)=\int_n^{n+1}\dfrac{\ud t}{n+1}-\int_n^{n+1}\dfrac{\ud t}{t}\\ &&\\ &=&\dsp \int_n^{n+1}\underset{< 0,\text{ sur }[n,n+1[}{\underbrace{\left(\dfrac{1}{n+1}-\dfrac{1}{t}\right)}}\ud t < 0 \end{array} $$ Donc $\boxed{\left(u_n\right)_{n\geq 1} \text{ est strictement décroissante }}$.
  2. Soit $k\in \N^*$, on a, $$\forall t\in [k,k+1],\quad \dfrac{1}{k+1}\leq \dfrac{1}{t}\leq \dfrac{1}{k}\Longrightarrow\dfrac{1}{k+1}\leq \ln(k+1)-\ln(k)\leq \dfrac{1}{k}.$$ On en déduit, pour tout $n\in \N^*$ $$\dsum_{k=1}^{n}\dfrac{1}{k+1}\leq \dsum_{k=1}^{n}(\ln(k+1)-\ln(k))\leq \dsum_{k=1}^{n}\dfrac{1}{k}\Longrightarrow \ln(n+1)\leq \dsum_{k=1}^n\dfrac{1}{k}$$ ce qui donne, $\boxed{u_n\geq \ln(n+1)-\ln(n)>0}$.
  3. La suite $(u_n)_{n\geq 1}$ est décroissante minorée par $0$ donc $(u_n)$ converge vers $\ell \geq 0$.
(Fonction $\Gamma$) Correction

Soit $x\in \R_+^*$.

  1. Montrer que $\dsp\int_0^\infty t^{x-1}\ee^{-t}\ud t$ est convergente. On note dans la suite $\Gamma(x)=\dsp\int_0^\infty t^{x-1}\ee^{-t}\ud t$.
  2. Montrer (soigneusement) que $\Gamma(x+1)=x\Gamma(x)$. En déduire la valeur de $\Gamma(n+1)$ pour $n\in \N$.
Correction

  1. Pour $x>0$ fixé. $f:t\mapsto t^{x -1}\ee^{-t}$ est continue , positive sur $]0,+\infty[$. On a
    1. $t^2f(t)=t^{x+1}\ee^{-t}\tendvers{t}{\infty} 0$ donc $f(t)=\underset{t\rightarrow\infty}{\mathrm{o}}\left(\dfrac1{t^2}\right)$. Or $\dsp\int_1^\infty \dfrac{\ud t}{t^2}$ converge donc $\dsp\int_ 1^{+\infty}{f(t)}\ud t$ existe.
    2. $f(t)\underset{t\to 0^+}{\thicksim}t^{x-1}=\dfrac 1{t^{1-x}}$. Or $\dsp\int_0^1 \dfrac{\ud t}{t^{1-x}}$ converge puisque $1-x < 1$ (car $x>0$) donc par des équivalents pour les fonctions positives : $\dsp\int_0^1 f(t)\ud t$ converge.
    Finalement, $\Gamma(x)$ existe.
  2. Soit $x> 0$. Soit $0 < a < b < \infty$, on effectue une intégration par parties sur le segment $[a,b]$ : $$\left\{ \begin{array}{lcl} u(t)&=&t^x,\\ v(t)&=&-\ee^{-t} \end{array} \right. \quad \left\{ \begin{array}{lcl} u'(t)&=&xt^{x-1}\\ v'(t)&=&\ee^{-t} \end{array}\right. $$ Ce qui donne, $$ \dsp\int_a^b t^x\ee^{-t}\ud t=\left[-t^x\ee^{-t}\right]_{a}^{b}+x\dsp\int_a^b t^{x-1}\ee^{-t}\ud t$$ Comme $x>0$, les intégrales $\Gamma(x)$ et $\Gamma(x+1)$ convergent et on a $\dsp\lim_{a\to 0^+}a^x\ee^{-a}=0$ et $\dsp\lim_{b\to+\infty}b^x\ee^{-b}=0$ donc on obtient après passages aux limites $\Gamma(x+1)=0-0+x\Gamma(x)$. $$ \boxed{\forall x>0,\quad\Gamma(x+1)=x\Gamma(x)}. $$ Soit $n\in \N^*$. On a $\Gamma(n+1)=n\Gamma(n)$ donc par récurrence sur $n\geq 1$, $\Gamma(n+1)=n!\Gamma(1)$. Or $$\Gamma(1)=\dsp\int_0^{+\infty}{\ee^{-t}}\ud t=\lim_{b\to+\infty}1-\ee^{-b}=1\Longrightarrow\boxed{\forall n\in \N^*, \Gamma(n+1)=n!}.$$ Relation reste vraie pour $n=0$ puisque $\Gamma(1)=1=0!$.
(Fonction $\Gamma$ Bis) Correction

Soit $x\in \R_+^*$. On pose $$\Gamma (x)=\int_0^\infty t^{x-1}\ee^{-t}\ud t.$$

  1. Montrer (soigneusement) que $\Gamma$ est continue sur $\R_+$.
  2. Montrer que $\Gamma\in \CC^\infty(\R_+^*)$ et déterminer $\Gamma^{(n)}$ pour tout $n\in \N$.
Correction

  1. On définit $\fonct{f}{(\R_+^*)^2}{\R}{(x,t)}{t^{x-1}\ee^{-t}}$.
    1. Il est clair que $f\in \CC^\infty((\R_+^*)^2)$ comme la composées et le produit des fonctions de classe $\CC^\infty$.
    2. Soient $0 < a < 1< < b< \infty $ deux réels, (les cas $[a,b]\subset ]0,1]$ ou $[a,b]\subset [1,\infty[$ sont simailaires) on pose $$ g(t)=\left\{\begin{array}{lcl} t^{a-1}\ee^{-t} &\text{ si }& t \in ]0,1[\\ &&\\ t^{b-1}\ee^{-t}&\text{ si }& t\gt 1 \end{array}\right. $$ Alors, $$\forall x\in [a,b],\quad \forall t\in \R_+^*, \quad 0\leq f(x,t)\leq g(t).$$ La fonction $g$ est continue par morceaux sur $\R_+^*$, de plus, $\\$ Au voisinage de $0$, $g(t)\sim t^{a-1}$ et $\dsp \int_0^1t^{a-1}\ud t $ converge, donc $\dsp \int_0^1g(t)\ud t $ converge.$\\$ Au voisinage de $\infty$, on a $t^2g(t)\tendvers{t}{\infty}0$, donc $g(t)\underset{t\to \infty}{=}\mathrm{o}\left(\frac{1}{t^2}\right)$.
    Donc d'après le cours $\Gamma\in \CC([a,b])$, ceci étant vrai pour tout segment de $\R_+^*$, on en déduit alors que $\Gamma \in \CC(\R_+^*)$.
  2. La fonction $f$ est de classe $\CC^\infty$ sur son domaine de définition, de plus: $$\forall k\in \N^*,\forall x > 0,\quad \dfrac{\partial^k \, f}{\partial x^k} (x,t) =\ln(t)^k t^{x-1}\ee^{-t}.$$ Soient $0 < a < 1< b< \infty $ deux réels et $k\in \N^*$ (les cas $[a,b]\subset ]0,1]$ ou $[a,b]\subset [1,\infty[$ sont simailaires), on a $$\forall x\in [a,b],\forall t\in ]0,1],\quad \abs{\dfrac{\partial^k \, f}{\partial x^k} (x,t)}= \abs{\ln(t)}^k t^{x-1}\ee^{-t} \leq t^{a-1}\abs{\ln(t)}^k\ee^{-t}.$$ $$\forall x\in [a,b],\forall t\in ]1,\infty[,\quad \abs{\dfrac{\partial^k \, f}{\partial x^k} (x,t)}= \ln(t)^k t^{x-1}\ee^{-t} \leq t^{b-1}\ln(t)^k\ee^{-t}.$$ on pose alors $$ g_k(t)=\left\{\begin{array}{lcl} t^{a-1}\abs{\ln(t)}^k\ee^{-t} &\text{ si }& t \in ]0,1[\\ &&\\ t^{b-1}\ln(t)^k\ee^{-t}&\text{ si }& t\gt 1 \end{array}\right. $$ La fonction $g_k$ est continue par morceaux sur $\R_+^*$, de plus, $\\$ Au voisinage de $0$, $$g_k(t)=t^{a/2}t^{-a/2} t^{a-1}\abs{\ln(t)}^k\ee^{-t} = t^{a/2-1} \left( t^{a/2}\abs{\ln(t)}^k\right) \ee^{-t}.$$ donc $t^{1-a/2}g_k(t) \tendvers{t}{0^+}\,0$ i.e. $g_k(t)\underset{t\to 0^+}{=} \mathrm{o}\left(\frac{1}{t^{1-a/2}}\right)$. Comme $\dsp \int_0^1 \dfrac{\ud t}{t^{1-a/2}} $ converge , donc $\dsp \int_0^1g_k(t)\ud t $ converge.$\\$ Au voisinage de $\infty$, on a $t^2g_k(t)\tendvers{t}{\infty}0$, donc $g_k(t)\underset{t\to \infty}{=}\mathrm{o}\left(\frac{1}{t^2}\right)$.$\\$ Ainsi, $g_k$ est intégrable sur $\R_+^*$. Ceci étant vrai pour tout $k\in \N^*$, alors d'après le cours $\Gamma\in \CC^\infty ([a,b])$.$\\$ Le résultat précédent est vrai pour tout segment $[a,b]$ inclus dans $\R_+^*$. On en déduit alors $$\Gamma\in \CC^\infty (\R_+^*),\quad \forall x > 0,\,\, \Gamma^{(k)}(x)=\int_0^\infty \ln(t)^k t^{x-1}\ee^{-t}\ud t.$$

Lien avec les séries

(Formule de Gauss) Correction

Soit $x\in \R_+^*$, on pose pour $n\geq 1$, $\gamma_n(x)=\dsp\int_0^n\left(1-\frac{t}{n}\right)^nt^{x-1}\ud t$

  1. Montrer que $\gamma_n(x)$ est bien définie.
  2. Montrer que la suite $(\gamma_n(x))$ converge dans $\R$ et que $\gamma_n(x)\tendversN\,\Gamma(x)$.
  3. En effectuant des intégrations par parties, montrer que $$\gamma_n(x)=\dfrac{n^x\,n!}{x(x+1)\cdots (x+n-1)(x+n)}=\dfrac{\ee^{-x\,u_n}}{x\dsp\prod_{k=1}^n\left(1+\frac{x}{k}\right)\ee^{-x/k}}.$$
  4. En déduire la formule de Gauss, $$\forall x >0,\quad \Gamma (x)=\lim_{n\to \infty}\dfrac{n^x\,n!}{x(x+1)\cdots (x+n-1)(x+n)}.$$
  5. On admet qu'il existe $\delta\in \R$ tel que, pour tout $x>0$, $\dfrac{1}{\Gamma(x)}=x\ee^{x\delta}\limiteX{n}{\infty}\dsp\prod_{k=1}^n\left(1+\frac{x}{k}\right)\ee^{-x/k}$. Quelle est le lien entre $\delta$ et la limite de la suite $(u_n)_{n\in \N^*}$.
Correction

  1. Soient $x>0$ et $n\in \N^*$. $f_n:t\mapsto \left(1-\frac{t}{n}\right)^n t^{x -1}$ est continue et positive sur $]0,n]$. On a $f_n(t)\underset{t\to 0^+}{\thicksim}t^{x-1}=\dfrac 1{t^{1-x}}$. Or $\dsp\int_0^1 \dfrac{\ud t}{t^{1-x}}$ converge puisque $1-x < 1$ (car $x>0$) donc par des équivalents pour les fonctions positives : $\dsp\int_0^1 f_n(t)\ud t$ converge. Finalement, $\gamma_n(x)$ existe.
  2. Pour $n\in \N^*$, on définit $h_n:]0,\infty[\longmapsto \R$ par $h_n(t)=\left\{\begin{array}{lcl} f_n(t)&\text{ si }& t\in ]0,n]\\ 0&\text{ si }&t>n \end{array}\right.$. On a
    1. $h_n$ est continue par morceaux sur $\R_+^*$
    2. Pour tout $t>0$, $h_n(t)\underset{n\to\infty}{=}\ee^{n\ln\left(1-\frac{t}{n}\right)}t^{x-1}\underset{n\to \infty}{\thicksim}\ee^{-t}t^{x-1}=h(t)$, donc $h_n\overset{C.S}{\tendversN}h$.
    3. Pour tout $n\geq 1$ et $t>0$, on a $0\leq h_n(t)\leq h(t)$, car si $t> n$ alors $h_n(t)=0\leq h(t)$, et pour $t\in ]0,n]$, on a $$n\ln\left(1-\frac{t}{n}\right)\leq n\frac{-t}{n}=-t, (\text{car } \forall x>-1, \,\ln(1+x)\leq x)$$ donc $\ee^{n\ln\left(1-\frac{t}{n}\right)}\leq \ee^{-t}.$ De plus $\dsp\int_0^\infty h(t)\ud t$ existe (hypothèse de domination).
    En appliquant le théorème de convergence dominée, on trouve: $$\gamma_n(x)=\int_0^\infty h_n(t)\ud t \tendversN \int_0^\infty h(t)\ud t=\int_0^\infty \ee^{-t}t^{x-1}\ud t=\Gamma(x).$$ On en déduit, $(\gamma_n(x))_n$ converge dans $\R$ et $\limiteX{n}{\infty}\gamma_n(x)=\Gamma(x)$.
  3. Soit $n\geq 1$ et $x>0$, on effectue une intégration par parties $$\left\{\begin{array}{ll} u(t)&=\left(1-\frac{t}{n}\right)^n,\\ & \\ v'(t)&=t^{x-1} \end{array}\right. \left\{\begin{array}{ll} u'(t)&=-\left(1-\frac{t}{n}\right)^{n-1}\\ &\\ v(t)&=\frac{t^x}{x} \end{array}\right. $$ Ce qui donne $$\dsp\gamma_n(x)=\left[\left(1-\frac{t}{n}\right)^n\frac{t^x}{x}\right]_{0}^n+\dfrac{1}{x}\dsp\int_0^n \left(1-\frac{t}{n}\right)^{n-1}t^{x}\ud t=\dfrac{1}{x}\int_0^n \left(1-\frac{t}{n}\right)^{n-1}t^{x}\ud t$$ On effectue encore une IPP en dérivant $\left(1-t/n\right)^{n-1}$ et intégrant $t^x$, $$ \begin{array}{lcl} \gamma_n(x)&= &\dsp 0 +\dfrac{1}{x}\int_0^n \left(1-\frac{t}{n}\right)^{n-1}t^{x}\ud t\\ &&\\ &=&\dsp\dfrac{1}{x}\left[\left(1-\frac{t}{n}\right)^{n-1}\frac{t^{x+1}}{x+1}\right]_{0}^n+\dfrac{1}{x(x+1)}\dfrac{n-1}{n}\dsp\int_0^n \left(1-\frac{t}{n}\right)^{n-2}t^{x+1}\ud t\\ &&\\ &=&\dsp \dfrac{1}{x(x+1)}\dfrac{n-1}{n}\dsp\int_0^n \left(1-\frac{t}{n}\right)^{n-2}t^{x+1}\ud t \end{array} $$ On continue, $$ \begin{array}{lcl} \gamma_n(x)&= &\dsp \dfrac{1}{x(x+1)(x+2)}\dfrac{n-1}{n}\dfrac{n-2}{n}\dsp\int_0^n \left(1-\frac{t}{n}\right)^{n-3}t^{x+2}\ud t\\ &&\\ &=&\dsp \dfrac{1}{x(x+1)(x+2)\cdots (x+n-1)}\dfrac{n-1}{n}\dfrac{n-2}{n}\cdots\dfrac{1}{n}\dsp\int_0^n t^{x+n-1}\ud t\\ &&\\ &=&\dsp \dfrac{1}{x(x+1)(x+2)\cdots (x+n-1)}\dfrac{n-1}{n}\dfrac{n-2}{n}\cdots\dfrac{1}{n} \left[\dfrac{t^{x+n}}{x+n}\right]_0^n \end{array} $$ D'où $$\boxed{\gamma_n(x)=\dfrac{1}{x(x+1)\cdots (x+n)}\dfrac{(n-1)!}{n^{n-1}}n^{n+x}=\dfrac{n!\,n^x}{x(x+1)\cdots (x+n)}}.$$ En écrivant, $(x+1)\cdots (x+n)=1\left(1+\frac{x}{1}\right) 2\left(1+\frac{x}{2}\right)\cdots n\left(1+\frac{x}{n}\right)=n! \dsp\prod_{k=1}^n\left(1+\frac{x}{k}\right)$, puis on multiplie la fraction par $\dfrac{\ee^{-x/1}\ee^{-x/2}\cdots\ee^{-x/n}}{\ee^{-x/1}\ee^{-x/2}\cdots\ee^{-x/n}}$, on obtient, $$\gamma_n(x)=\dfrac{n!\ee^{x\ln(n)} \ee^{-x/1}\ee^{-x/2}\cdots\ee^{-x/n}}{x\,n!\prod_{k=1}^n\left(1+\frac{x}{k}\right) \ee^{-x/1}\ee^{-x/2}\cdots\ee^{-x/n}}= \dfrac{\ee^{-x\ln(n)}\ee^{-x\sum\limits_{k=1}^n\frac{1}{x}}}{x\prod_{k=1}^n\left(1+\frac{x}{k}\right)\ee^{-\frac{x}{k}}}$$ Soit $$\boxed{\gamma_n(x)=\dfrac{\ee^{-x u_n}}{x\prod\limits_{k=1}^n\left(1+\frac{x}{k}\right)\ee^{-\frac{x}{k}}}}.$$
  4. En utilisant les résultats des deux questions précédentes, on trouve $$\forall x >0,\quad \Gamma (x)=\lim_{n\to \infty}\gamma_n(x)=\lim_{n\to \infty}\dfrac{n^x\,n!}{x(x+1)\cdots (x+n-1)(x+n)}.$$
  5. Notons, pour $n\in \N^*$ et $x>0$, $v_n(x)=\dsp\prod_{k=1}^n\left(1+\frac{x}{k}\right)\ee^{-x/k}$. On considère la série $\dsum \ln(v_{n+1}(x))-\ln(v_n(x))$, on a $$ \ln(v_{n+1}(x))-\ln(v_n(x))=\ln\left(\dfrac{v_{n+1}(x)}{v_n(x)}\right)=\ln\left(1+\frac{x}{n+1}\right)-\dfrac{x}{n+1}=\underset{n\to \infty}{\mathrm{O}}\left(\frac{x^2}{n^2}\right)$$ On en déduit que la série $\dsum \ln(v_{n+1}(x))-\ln(v_n(x))$ converge d'où $\ln(v_n)\tendversN \ell(x)$ ce qui implique que $v_n(x)\tendversN V(x)=\ee^{\ell (x)}>0$. Ce qui donne également $\dfrac{1}{v_n(x)}\tendversN \dfrac{1}{V(x)}$.$\\$ Comme on a, $\gamma_n(x)=\dfrac{\ee^{-xu_n}}{xv_n(x)}$, et étant donnée que les trois suites $\gamma_n(x),\,u_n$ et $v_n(x)$ admettent des limites finies, on en déduit, $$0<\Gamma(x)=\limiteX{n}{\infty}\gamma_n(x)=\limiteX{n}{\infty}\dfrac{\ee^{-xu_n}}{xv_n(x)}=\dfrac{\limiteX{n}{\infty} \ee^{-xu_n}}{\limiteX{n}{\infty}xv_n(x)}=\dfrac{\ee^{-x\ell}}{xV(x)}.$$ Ce qui donne $$\boxed{\dfrac{1}{\Gamma(x)}=\dfrac{xV(x)}{\ee^{-x\ell}}=x\ee^{x\ell}\limiteX{n}{\infty}\dsp\prod_{k=1}^n\left(1+\frac{x}{k}\right)\ee^{-x/k}}.$$ Donc $\boxed{\delta=\ell=\limiteX{n}{\infty}\left(\dsum_{k=1}^n\dfrac{1}{k} -\ln(n)\right)}$.
($\star\star$) Correction

Soit $n\in \N^*$. On définit $f_n$ sur $\R_+^*$ par $\dsp f_n(x)=\dfrac{n^x\,n!}{\dsp \prod_{\substack{0\leq k\leq n}}(x+k)}$.

  1. Prouver l'existence, pour tout $x>0$ de $\Gamma(x)= \limiteX{n}{\infty}f_n(x)$.
  2. Montrer l'existence d'une constante $\gamma$ telle que : $\dsp\forall x>0,\quad \ln (\Gamma(x))=-\ln(x)-\gamma x+\dsum_{n\geq 1} \left(\dfrac{x}{n}-\ln\left(1+\frac{x}{n}\right)\right).$
  3. Montrer que $\Gamma$ est de classe $\CC^1$ sur $\R_+^*$.

Correction

  1. Comme $f_n$ est strictement positive, on considère alors $\ln(f_n)$. Soit $x>0$, on a $$\ln(f_n(x))=x\ln(n)+\ln(n!)-\dsum_{k=0}^n\ln(x+k),$$ ce qui donne: $$\forall n\geq 2,\,\ln(f_{n}(x))-\ln(f_{n-1}(x))= -x\ln(1-\frac{1}{n})+\ln(n)-\ln(x+n).$$ Soit, $$\ln(f_{n}(x))-\ln(f_{n-1}(x))=\dfrac{x}{n}+\mathrm{O}\left(\frac{1}{n^2}\right) +\ln(n)-\ln(n)- \ln(1+\frac{x}{n})=\mathrm{O}\left(\frac{1}{n^2}\right)$$ Autrement dit, la série $\dsum_{n\geq 2} \left(\ln(f_n(x))-\ln(f_{n-1}(x))\right)< \infty$. On en déduit que $\ln(f_n(x))\tendversN \ell (x)\in \R$, soit $f_n(x)\tendversN\ee^{\ell (x)}=\Gamma(x)$.
  2. Soit $x>0$, pour tout $n\geq 1$, on a $$\begin{array}{lcl} \ln(f_n(x))&=&x\ln(x)+\ln(n!)-\dsum_{k=0}^n\ln(x+k)\\ &=&x\ln(n)+\dsum_{k=1}^n\ln(k)-\dsum_{k=0}^n\ln(x+k)\\ &=&x\ln(x)-\dsum_{k=1}^n\ln\left(1+\frac{x}{k}\right)-\ln(x)\\ &=&-\ln(x)+x\left(\ln(n)-\dsum_{k=1}^n\frac{1}{k}\right)+\dsum_{k=1}^n\left(\frac{x}{k}-\ln\left(1+\frac{x}{k}\right)\right) \end{array}$$ Or, la suite $\ln(n)-\dsum_{k=1}^n\frac{1}{k}$ converge dans $\R$ vers $-\gamma$ (avec $\gamma>0$) et la série $\dsum_{k=1}^n\left(\frac{x}{k}-\ln\left(1+\frac{x}{k}\right)\right)$ converge également puisque $\dfrac{x}{k}-\ln\left(1+\dfrac{x}{k}\right)=\underset{n\to\infty}{\mathrm{O}}\left(\frac{x}{n^2}\right)$.
    Comme $(\ln(f_n(x)))$ converge également dans $\R$ d'après la question précédente, on en déduit, $$\limiteX{n}{\infty}\ln(f_n(x))= \limiteX{n}{\infty}-\ln(x)+x\left(\ln(n)-\dsum_{k=1}^n\frac{1}{k}\right)+\dsum_{k=1}^n\left(\frac{x}{k}-\ln\left(1+\frac{x}{k}\right)\right),$$ ce qui donne, $$\ln(\Gamma(x))=-\ln(x)-\gamma x+\dsum_{k\geq 1}\left(\frac{x}{k}-\ln\left(1+\frac{x}{k}\right)\right).$$
  3. Il faut juste montrer que $x\longmapsto \dsum_{k\geq 1}\left(\frac{x}{k}-\ln\left(1+\frac{x}{k}\right)\right)$ est de classe $\CC^1$ sur $\R_+^*$. On note pour $n\geq 1$, $$\forall x\in \R_+,\quad f_n(x)= \frac{x}{n}-\ln\left(1+\frac{x}{n}\right).$$ On a $f_n\in \CC^1(\R_+)$ et $\dsum f_n $ converge simplement sur $\R_+^*$. De plus, $$\forall n\geq 1,\, x\geq 0, \quad f_n'(x)=\dfrac1n -\dfrac{1/n}{1+\frac{x}{n}}=\dfrac1n -\dfrac{1}{x+n}=\dfrac{x}n({x+n)}.$$ Soit $a>0$, on a $$\forall x\in [0,a],\quad \abs{f_n'(x)}\leq \dfrac{a}{n^2}\Longrightarrow \norme{f_n'}_\infty^{[0,a]}\leq \dfrac{a}{n^2}.$$ Comme $\dsum \dfrac{a}{n^2}$ converge, on en déduit que $\dsum f_n'$ converge normalement sur $[0,a]$ donc uniformement. D'après le cours, alors $\dsum f_n\in \CC^1([0,a])$. $a$ étant arbitraiment choisi, on en déduit alors, $$\dsum f_n\in \CC^1(\R_+),\quad \forall x\in \R_+,\quad \left(\dsum f_n\right)'(x)=\dsum_{n\geq 1}\dfrac{x}{n(x+n)}.$$ On en déduit alors que $\ln (\Gamma)\in \CC^1(\R_+^*)$, et que: $$\forall x>0,\quad \ln(\Gamma (x))'= -\dfrac{1}{x}-\gamma +\dsum_{n\geq 1}\dfrac{x}{n(x+n)}.$$

($\star\star$) Correction

On définit $f$ sur $\R_+^*$ par $f(x)=\dfrac{\Gamma' (x)}{\Gamma (x)}$

  1. Montrer que, pour tout $x>0$, $$f(x)= -\dfrac1x -\gamma +\dsum_{n\geq 1}\dfrac{x}{n(x+n)}$$
  2. Montrer que, pour tout $x>0$, $$\dfrac{\Gamma''(x)\Gamma(x)-\Gamma'^2(x)}{\Gamma^2(x)}=\dsum_{n\geq 0}\dfrac{1}{(n+x)^2}$$
  3. Montrer que $g=\ln \circ \Gamma $ est convexe.

Correction

  1. On a déjà montré dans l'exercice précédente que $\ln\circ \Gamma \in \CC^1 (\R_+^*)$ et que : $$\forall x>0,\quad \ln(\Gamma (x))'= -\dfrac{1}{x}-\gamma +\dsum_{n\geq 1}\dfrac{x}{n(x+n)}.$$ Or, $\Gamma \in \CC^1(\R_+^*)$ donc en utilisant les régles de dérivation, on obtient: $$\forall x>0,\quad \ln(\Gamma (x))'=\dfrac{\Gamma'(x)}{\Gamma(x)}=- \dfrac{1}{x}-\gamma +\dsum_{n\geq 1}\dfrac{x}{n(x+n)}.$$
  2. Notons, pour $n\geq 1$, $g_n(x) = \dfrac{x}{n(x+n)}$, on a :$\\$ $\mathbf{a)}$ $\dsum g_n$ converge simplement sur $\R_+^*$.$\\$ $\mathbf{b)}$ Pour tout $n\geq 1$, $g_n\in \CC^1(\R_+^*)$ et $$\forall x>0,\quad g_n(x)=\dfrac{n(x+n)-nx}{n^2(x+n)^2}=\dfrac{1}{(x+n)^2}.$$ $\mathbf{c)}$ $\sum g_n'$ converge normalement sur $\R_+^*$ donc uniformement, puisque $\norme{g_n'}_\infty\leq \dfrac{1}{n}$. $\\$ On en déduit que $\dsum g_n$ est de classe $\CC^1$ sur $\R_+^*$, de plus , $$\forall x>0,\quad \left(\dsum_{n\geq 1}g_n\right)'(x)=\dsum_{n\geq 1}\dfrac{1}{(x+n)^2}.$$ Comme $x\longmapsto \dfrac1x\in \CC^1(\R_+^*)$, on en déduit que $f\in \CC^1(\R_+^*)$ et $$\forall x>0,\quad f'(x)=\dfrac{1}{x^2}+ \dsum_{n\geq 1}\dfrac{1}{(x+n)^2}=\dsum_{n\geq 0}\dfrac{1}{(x+n)^2}.$$ D'autre part, $\Gamma \in \CC^\infty(\R_+^*)$, donc en dérivant directement $f$, on obtient: $$\forall x>0,\quad f'(x)=\dfrac{\Gamma''(x)\Gamma(x)-\Gamma'^2(x)}{\Gamma^2(x)}.$$ Ce qui donne : $$\boxed{\forall x>0,\quad \dfrac{\Gamma''(x)\Gamma(x)-\Gamma'^2(x)}{\Gamma^2(x)}=\dsum_{n\geq 0}\dfrac{1}{(x+n)^2}}.$$
  3. La fonction $x\longmapsto \ln(\Gamma (x))$ est de classe $\C^2$ sur $\R_+^*$, de plus, $$\forall x>0,\, (\ln(\Gamma))''(x)=\dfrac{\Gamma''(x)\Gamma(x)-\Gamma'^2(x)}{\Gamma^2(x)}= \dsum_{n\geq 0}\dfrac{1}{(x+n)^2}>0$$ donc $\ln \circ \Gamma$ est convexe.

Equivalence

(Mines PSI 2017) Correction

On définit $f$ sur $\R_+^*$ par $f(x)=\dsp\int_x^{x+1}\ln(\Gamma(t))\ud t$.

Trouver un équivalent de $f$ lorsque $x$ tend vers $\infty$.

Correction

La fonction $f$ est bien définie, de plus la fonction $h$ définie sur $\R_+^*$ par $h(t)=\ln(\Gamma(t))$ est continue donc il admet une primitive $H$, ainsi, on peut écrire: $$\forall x\gt 0,\quad f(x) =\int_x^{x+1}\ln(\Gamma(t))\ud t= H(x+1)-H(x).$$ Ainsi, $f$ est de classe $\CC^1$ et on a : $$\forall x> 0,\quad f'(x)=H'(x+1)-H'(x)=\ln(\Gamma (x+1))-\ln(\Gamma (x) ) =\ln(x).$$ Puisque $\ln(\Gamma (x+1))=\ln(x\Gamma (x)) =\ln(x)+\ln (\Gamma(x))$.$\\$ Ainsi, il existe $\alpha\in \R$ tel que $$\forall x\in \R,\quad f(x)= \int_x^{x+1}\ln(\Gamma(t))\ud t = x\ln(x)-x +\alpha.$$ Notons que $\dsp\int_0^1 \ln(\Gamma (t))\ud t $ converge car $\Gamma (x)\underset{x\to 0^+}{\sim }\dfrac{1}{x}$, donc $\ln(\Gamma (x))\underset{x\to 0^+}{\sim }-\ln(x)$ et on sais que $\dsp\int_0^1\ln(t)\ud t $ converge. $\\$ En faisant $x$ tendre vers $0^+$, on trouve $$\forall x\in \R,\quad f(x)= \int_x^{x+1}\ln(\Gamma(t))\ud t = x\ln(x)-x +\int_0^1\ln(\Gamma (t))\ud t.$$ On peut montrer que $\dsp \int_0^1\ln(\Gamma (t))\ud t= \dfrac{\ln(2\pi)}{2} $.$\\$ Finalement, $$\boxed{f(x) \underset{x\to \infty}{\sim }x\ln(x)}.$$

Equivalence Correction

Montrer la relation suivante: $$\Gamma(x)\underset{x\to \infty}{\sim }\left(\dfrac{x}{\ee}\right)^x \sqrt{\dfrac{2\pi}{x}}.$$

Correction

On pose pour $x> 0$, $h(x)=\ln(\Gamma (x))$ et $f(x)=\dsp\int_x^{x+1}h(t)\ud t$, En faisant une IPP, on a $$ f(x)=\left[(t-x-\frac 1 2)h(t)\right]_x^{x+1}-\int_x^{x+1} (t-x-\frac 1 2)h'(t)\ud t =\dfrac{h(x)+h(x+1)}{2} -\int_x^{x+1} (t-x-\frac 1 2)h'(t)\ud t.$$ puis, $$ \begin{array}{lcl} \dsp \int_x^{x+1} (t-x-\frac 1 2)h'(t)\ud t&= &\dsp \left[\dfrac{(t-x)^2-(t-x)}{2}h'(t)\right]_x^{x+1}-\int_x^{x+1} \dfrac{(t-x)^2-(t-x)}{2}h''(t)\ud t\\ &&\\ &=&\dsp -\int_x^{x+1} \dfrac{(t-x)^2-(t-x)}{2}h''(t)\ud t \end{array}$$ donc $$f(x)=\dfrac{h(x)+h(x+1)}{2} - \dfrac 1 2\int_x^{x+1} (t-x)(1-(t-x))h''(t)\ud t.$$ Or, pour $t\in [x,x+1]$, on a $0\leq (t-x)(1-(t-x))\leq \dfrac 1 4$, d'autre part, la fonction $h''\geq 0$, donc $$0\leq \int_x^{x+1} (t-x)(1-(t-x))h''(t)\ud t \leq \dfrac 1 4 \int_x^{x+1}h''(t)\ud t =\dfrac{h'(x+1)-h'(x)}{4}=\dfrac{\Gamma'(x+1)}{\Gamma (x+1)}-\dfrac{\Gamma'(x)}{\Gamma (x)}$$ En utilisant la relation $\Gamma (x+1)=x\Gamma (x)$, on trouve : $$0\leq \int_x^{x+1} (t-x)(1-(t-x))h''(t)\ud t \leq \dfrac{1}{4x}$$ d'après l'exercice (Mines 2017), on a : $$\forall x\in \R,\quad f(x) = x\ln(x)-x +\dfrac{\ln(2\pi)}{2}.$$ on a aussi, $$ \dfrac{h(x)+h(x+1)}{2} =\dfrac{\ln(\Gamma (x+1))+\ln(\Gamma (x))}{2}= \dfrac{\ln(x\Gamma (x))+\ln(\Gamma (x))}{2}=\ln(\Gamma (x))+\ln (\sqrt{x})$$ Ce qui donne finalement, pour tout $x> 0$, $$ 0 \leq \ln(\Gamma (x)) +\ln(\sqrt{x}) -f(x) \leq \dfrac{1}{8x}$$ soit $$ 0 \leq \ln(\Gamma (x)) +\ln(\sqrt{x}) -\ln\left(\left(\frac{x}{ \ee}\right)^x \sqrt{2\pi}\right) \leq \dfrac{1}{8x}$$ Ensuite, $$ 1\leq \dfrac{\Gamma(x)\sqrt{x}}{\sqrt{2\pi}}\left(\dfrac{\ee}{ x}\right)^x \leq \ee^{1/(8x)}$$ En faisant $x$ tendre vers $\infty$, on trouve la relation demandée.

CCINP 2024 Correction

Pour $k\in \N^*$, on définit $$\rho_k=\ln(k)-\int_{k-1/2}^{k+1/2}\ln(t)\ud t$$

  1. Montrer que pour tout $n\in \N^*$, on a $$\ln(\Gamma (n)) = \int_{1/2}^{n-1/2}\ln(t)\ud t +\dsum_{k=1}^{n-1}\rho_k.$$
  2. Montrer que $$\forall k\geq 1,\quad \rho_k=\int_0^{1/2}-\ln\left(1-\dfrac{t^2}{k^2}\right)\ud t.$$ En déduire que $\dsum \rho_k$ converge.
  3. Montrer qu'il existe $c\in \R$, tel que $$\ln(\Gamma(n))=\left(n-\frac 1 2\right)\ln(n) -n+c+\underset{n\to \infty}{\mathrm{o}}(1).$$ Puis, $$\Gamma (n) \underset{n\to \infty}{\sim }\ee^c n^{n-1/2}\ee^{-n}.$$
Correction

  1. Montrons cette relation par récurrence sur $n\in \N^*$.$\\$ Pour $n=1$, on a $\ln(\Gamma(1))=\ln(1)=0$, d'autre part $$\int_{1/2}^{1-1/2}\ln(t)\ud t +\dsum_{k=1}^{1-1}\rho_k = \int_{1/2}^{1/2}\ln(t)\ud t +0=0$$ donc la relation est vrai pour $n=1$.$\\$ Soit $n\geq 1$ et supposons que la relation est vrai au range $n$, on a alors $$ \begin{array}{lclr} \ln(\Gamma (n+1))&=&\ln(n\Gamma (n))=\ln (n)+\ln (\Gamma (n))&\\ &&&\\ &=&\dsp \ln (n)+\int_{1/2}^{n-1/2}\ln(t)\ud t +\dsum_{k=1}^{n-1}\rho_k&\text{ (HR)}\\ &&&\\ &=&\dsp \ln (n)+\int_{1/2}^{n-1/2}\ln(t)\ud t +\dsum_{k=1}^{n-1}\rho_k \textcolor{blue}{+\rho_n-\rho_n}&\\ &&&\\ &=&\dsp \ln (n)+\int_{1/2}^{n-1/2}\ln(t)\ud t +\dsum_{k=1}^{n}\rho_k -\ln(n)+\int_{n-1/2}^{n+1/2}\ln(t)\ud t&\\ &&&\\ &=&\dsp \int_{1/2}^{n+1-1/2}\ln(t)\ud t +\dsum_{k=1}^{n+1-1}\rho_k & (n+1/2 =n+1-1/2 ) \end{array} $$ Donc la relation est vraie au range $n+1$, ce qui prouve finalement que la relation est vraie pour tout $n\in \N^*$.
  2. Soit $k\geq 1$, on a $$ \begin{array}{lcl} \rho_k&=&\dsp \ln(k)-\int_{k-1/2}^{k+1/2}\ln(t)\ud t\\ &=& \dsp \int_0^{1/2}(2\ln(k))\ud t -\int_{k-1/2}^k\ln(t)\ud t -\int_k^{k+1/2}\ln(t)\ud t\\ &=& \dsp \int_0^{1/2}(2\ln(k))\ud t -\int_{1/2}^0\ln(k-u)(-\ud u) -\int_0^{1/2}\ln(k+u)\ud u\\ &=&\dsp \int_0^{1/2}(2\ln(k))\ud t -\int_0^{1/2}\ln(k-t)(\ud t) -\int_0^{1/2}\ln(k+t)\ud t\\ &=&\dsp \int_0^{1/2}\left(\ln\left(k^2\right)- \ln\left(k^2-t^2\right)\right)\ud t \\ &=&\dsp -\int_0^{1/2}\ln\left(\dfrac{k^2-t^2}{k^2}\right)\ud t =-\int_0^{1/2}\ln\left(1-\frac{ t^2}{ k^2}\right)\ud t \end{array} $$ La fonction $t\longmapsto -ln\left(1-\frac{t^2}{k^2}\right)$ est strictement croissante sur $[0,1/2]$, on en déduit alors : $$\forall k\geq 1,\quad 0\leq \rho_k\leq - \int_0^{1/2}\ln\left(1-\frac{ (1/2)^2}{ k^2}\right)\ud t \leq \dfrac{-1}{2}\ln\left(1-\frac{ 1}{ 4k^2}\right). $$ Or $$ \dfrac{-1}{2}\ln\left(1-\frac{ 1}{ 4k^2}\right) = \dfrac{1}{8k^2} +\underset{k\to \infty}{\mathrm{O}}(1/k^2)$$ Comme $\dsum \dfrac{1}{k^2}$ converge, on en déduit alors que $\dsum \rho_k$ converge, notons dans la suite $\ell =\dsum_{k=1}^\infty \rho_k$.
  3. En utilisant la question 1, on a $$ \ln(\Gamma(n)) =\left[t\ln(t)-t\right]_{1/2}^{n-1/2}+\dsum_{k=1}^{n-1}\rho_k = (n-1/2)\ln(n-1/2)-(n-1/2)-\dfrac{\ln(1/2)}{2} +1/2+\dsum_{k=1}^{n-1}\rho_k. $$ puis, $$\begin{array}{lcl} \ln(\Gamma(n))&=&(n-1/2)\ln(n) +(n-1/2)\ln\left(1-\frac{1}{2n} \right) -(n-1/2) -\dfrac{\ln(1/2)}{2} +1/2+\ell -\dsum_{k\geq 0}\rho_k\\ &&\\ &=& (n-1/2)\ln(n) + (n-1/2) \left(-\dfrac{1}{2n}+\underset{n\to \infty}{\mathrm{o}_1}(1/n)\right) -(n-1/2) -\dfrac{\ln(1/2)}{2} +1/2+\ell -\dsum_{k\geq 0}\rho_k\\ &&\\ &=&(n-1/2)\ln(n)-n +\underbrace{(\frac{\ln(2)+1+2\ell}{2})}_{c} + \underbrace{\mathrm{o}_1(1)-\dsum_{k\geq n}\rho_k}_{\mathrm{o}(1)} \end{array} $$ Puisque $\dsum \rho_k$ converge, alors $\dsum_{k\geq n}\rho_k\tendversN\,0$, donc finalement, on a $$\ln(\Gamma(n))=\left(n-\frac 1 2\right)\ln(n)-n+c +\underset{n\to \infty}{\mathrm{o}}(1).$$ Donc $${\ln(\Gamma(n))}-{\left(n-\frac 1 2\right)\ln(n)-n+c} \tendversN\,0$$ Par continuité de la fonction $\exp$, on trouve : $$\ee^{ {\ln(\Gamma(n))}-{\left(n-\frac 1 2\right)\ln(n)-n+c}}=\dfrac{\Gamma(n)}{\ee^{\left(n-\frac 1 2\right)\ln(n)-n+c}}\tendversN\,1$$ Autrement dit, $$\boxed{\Gamma(n)\underset{n\to \infty}{\sim}\ee^{\left(n-\frac 1 2\right)\ln(n)-n+c}=\ee^c\ee^{-n}n^{n-1/2} }.$$

$\Gamma$ et $\zeta$

($\star\star$) Correction

Soit $x> 0$. On pose $F(x)=\dsp \int_0^\infty \dfrac{t^x}{\ee^{t}-1}\ud t$

  1. Montrer que $F(x)$ est bien définie.
  2. Montrer que $F(x)=\zeta(x+1)\Gamma (x+1)$. (pour la définition de la fonction $\zeta$ voir l'exercice fonction $\zeta$)
Correction

Soit $x> 0$. La fonction $\varphi (t)=\dfrac{t^x}{\ee^t-1}$ est continue sur $\R_+^*$. De plus, au voisinage de $0$, on a $\varphi(t)\underset{t\to 0^+}{\sim} t^{x-1} $ et $\dsp\int_0^1 t^{x-1}\ud t $ converge car $x-1 > -1$. $\\$ Au voisinage de $\infty$, on a $t^2\varphi(t) \tendvers{t}{\infty}\,0$ donc $\varphi (t)\underset{t\to \infty}{\sim}\mathrm{o}(1/t^2)$ et d'après le cours $\dsp\int_1^\infty \dfrac{\ud t}{t^2}$ converge.$\\$ On en déduit alors que $F(x)$ est bien définie. $\\$ Soit $tgt 0$ alors $\ee^{-t}\in ]0,1[$, on peut alors écrire: $$ \varphi(t) = \dfrac{t^x}{\ee^t -1} =\dfrac{t^x\ee^{-t}}{1-\ee^{-t}}=\dsum_{n\geq 0} t^x\ee^{-t}\ee^{-nt}=\dsum_{n\geq 0}\varphi_n(t)$$ avec $\varphi_n(t)=t^x\ee^{-(n+1)t}$. On a :$\\$ $\quad \mathbf{a)}$ Pour tout $n\in \N$, $\varphi_n\in \CC(\R_+^*)$, $\\$ $\quad \mathbf{b)}$ $\dsum \varphi_n$ converge simplement sur $\R_+^*$ vers $\varphi$, $\\$ $\quad\mathbf{c)}$ Pour $n\in \N$, on a $$\norme{\varphi_n}_1=\int_0^\infty t^x\ee^{-(n+1)t}\ud t =\int_0^\infty \left(\frac{u}{n+1}\right)^x\ee^{-u}\dfrac{\ud u}{n+1} = \dfrac{\Gamma(x+1)}{n^{x+1}}$$ et $\dsum \dfrac{1}{n^{x+1}}$ converge car $x > 0$, donc $\dsum \norme{\varphi_n}_1$ converge.$\\$ On en déduit d'après le cours, $$\boxed{\int_0^\infty\varphi(t)\ud t =\dsum_{n\geq 0} \int_0^\infty\varphi_n(t)\ud t = \Gamma(x+1)\zeta (x+1)}.$$

$\Gamma$ et $\beta$

(Fonction bêta d'Euler) Correction

Soient $x,y\in \R_+^*$, on définit $$B(x,y)=\int_0^1t^{x-1}(1-t)^{y-1}\ud t$$

  1. Montrer que $B$ est bien définie
  2. Montrer les deux relations suivantes: $$B(x,y)=B(y,x),\quad B(x+1,y)=\dfrac{x}{x+y}B(x,y)$$
  3. Montrer que $$B(x,y)=\dfrac{\Gamma (x)\Gamma (y)}{\Gamma(x+y)}.$$
Correction

  1. Soient $x,y\in \R_+^*$, la fonction $f(t)=t^{x-1}(1-t)^{y-1}$ est continue sur $]0,1[$. De plus $f(t)\underset{t\to 0^+}{\sim} t^{x-1}$ et $f(t)\underset{t\to 1^-}{\sim} (1-t)^{y-1}$, comme les deux intégrales $\dsp \int_0^1 t^{x-1}\ud t$ et $\dsp\int_0^1(1-t)^{y-1}\ud t$ sont convergentes, on en déduit alors que $B(x,y)$ est bien défini.
  2. Soient $x,y\in \R_+^*$. On effectue le changement de variable $u=1-t$ dans l'intégrale définissant $B(x,y)$, ce qui donne: $$B(x,y)=\int_1^0 (1-u)^{x-1} u^{y-1} (-\ud u) =\int_0^1u^{y-1} (1-u)^{x-1}=B(y,x).$$ Pour la deuxième relation, on fait une IPP: $$ \begin{array}{lcl} B(x+1,y)&=& \dsp \int_0^1 t^{x}(1-t)^{y-1}\ud t=\int_0^1 \left(\frac{t}{1-t}\right)^{x} (1-t)^{y+x-1}\ud t\\ &&\\ &=&\dsp \left[\left(\dfrac{t}{1-t}\right)^{x} \dfrac{-(1-t)^{x+y}}{x+y}\right]_0^1+\dfrac{x}{x+y}\int_0^1 \left(\dfrac{t}{1-t}\right)^{x-1}\dfrac{1}{(1-t)^2}(1-t)^{x+y}\ud t\\ &&\\ &=& \dsp \dfrac{x}{x+y}\int_0^1 t^{x-1}(1-t)^{y-1}\ud t\\ &&\\ &=& \dfrac{x}{x+y}B(x,y) \end{array} $$
  3. On rappel la formule de Gausse, $$\forall x> 0,\quad \Gamma(x) =\lim_{n\to \infty} \underbrace{\dfrac{n^x\,n!}{x(x+1)\cdots (x+n)}}_{\gamma_n(x)}$$ Soient $x,y\in \R_+^*$. En utilisant la question précédente, $$B(x+2,y) =\dfrac{x+1}{x+y+1}B(x+1,y)=\dfrac{x(x+1)}{(x+y)(x+y+1)}B(x,y)$$ Puis, $$B(x+3,y)=\dfrac{x+2}{x+y+2}B(x+2,y)= \dfrac{x(x+1)(x+2)}{(x+y)(x+y+1)(x+y+2)}B(x,y)$$ Ainsi, par réucrrence sur $n\in \N$, $$B(x+n+1,y)=\dfrac{x(x+1)\cdots (x+n)}{(x+y)(x+y+1)\cdots (x+y+n)}B(x,y)$$ On peut reécrire, sous la forme: $$B(x+n+1,y)=\dfrac{x\cdots (x+n)}{n^x \,n!}\dfrac{n^{(x+y)}\,n!}{(x+y)\cdots (x+y+n)}\dfrac{1}{n^y} B(x,y)$$ Ainsi, $$\tag{$\star$} B(x,y)=\dfrac{\gamma_n(x)}{\gamma_n(x+y)}n^yB(x+n+1,y)=\dfrac{\gamma_n(x)}{\gamma_n(x+y)}n^y\int_0^1t^{x+n}(1-t)^{y-1}\ud t $$ Ensuite, on a $$ n^y\int_0^1t^{x+n}(1-t)^{y-1}\ud t =n^y\int_0^1 (1-t)^{x+n}t^{y-1} \underset{u=nt}{=}\int_0^n \left(1-\frac{u}{n}\right)^{x+n}(u)^{y-1}\ud u $$ On définit alors: $$f_n(u)=\left\{\begin{array}{ll} \left(1-\frac{u}{n}\right)^{x+n}(u)^{y-1}& \text{ si } u\in ]0,n[\\ &\\ 0 & \text{ si } u\in [n,\infty[ \end{array} \right. $$ On a $f_n \overset{C.S}{\tendversN\,} f$ avec $f(u)=\ee^{-t}t^{y-1}$ et pour tout $n\geq 0$, $f_n\leq f$, de plus $\dsp \int_0^\infty f$ converge (voir aussi correction de l'exercice Formule de Gauss Alors, d'après le théorème de convergence domainé, on a : $$n^y B(x+n+1,y)=\int_0^\infty f_n(t)\ud t \tendversN\, \int_0^\infty f(t)\ud t =\Gamma (y).$$ On en déduit Finalement, en passant à la limite dans la relation ($\star$), $$B(x,y)= \dfrac{\Gamma(x)\Gamma (y)}{\Gamma (x+y)}.$$